The Student Room Group

Easy differential equation

The question asks to solve the ODE dydx=1x21 \displaystyle \frac{dy}{dx}=\frac{1}{x^2-1} given that y(0)=0 y(0)=0 .
Is the solution 12lnx1x+1 \displaystyle \frac{1}{2} \ln \left | \frac{x-1}{x+1} \right | which would be valid for x±1 x\neq \pm 1 OR
Is the solution just 12lnx1x+1 \displaystyle \frac{1}{2} \ln \frac{x-1}{x+1} which is valid for |x|<1?
Reply 1
Anyone?
Original post by Ano123
The question asks to solve the ODE dydx=1x21 \displaystyle \frac{dy}{dx}=\frac{1}{x^2-1} given that y(0)=0 y(0)=0 .
Is the solution 12lnx1x+1 \displaystyle \frac{1}{2} \ln \left | \frac{x-1}{x+1} \right | which would be valid for x±1 x\neq \pm 1 OR
Is the solution just 12lnx1x+1 \displaystyle \frac{1}{2} \ln \frac{x-1}{x+1} which is valid for |x|<1?


(1x21)dx=(11x2)dx=artanh(x)+c\int( \frac{1}{x^2 - 1})dx = -\int(\frac{1}{1-x^2})dx= -artanh(x) +c

So the solution is negative of what you have and it is valid for x<1 |x|<1
(edited 7 years ago)
Reply 3
Original post by NotNotBatman
(1x21)dx=(11x2)dx=artanh(x)+c\int( \frac{1}{x^2 - 1})dx = -\int(\frac{1}{1-x^2})dx= -artanh(x) +c

So the solution is negative of what you have and it is valid for x<1 |x|<1


So in logarithmic form do we still need the absolute value in the log?
Original post by Ano123
So in logarithmic form do we still need the absolute value in the log?


No. 12ln(1+x1x)=12[ln(1+x)ln(1x)]\displaystyle -\frac{1}{2} \ln (\frac{1+x}{1-x}) = -\frac{1}{2}[ln(1+x) - ln(1-x)] Hopefully you can see that any of x substituted such that 1<x<1 -1<x<1 or x<1|x|<1 will have a positive argument in the logs, so there is no need for the absolute value.
(edited 7 years ago)
Reply 5
Original post by NotNotBatman
No. 12ln(x1x+1)=12[ln(x1)ln(x+1)]\displaystyle -\frac{1}{2} \ln (\frac{x-1}{x+1}) = -\frac{1}{2}[ln(x-1) - ln(x+1)] Hopefully you can see that any of x substituted such that 1<x<1 -1<x<1 or x<1|x|<1 will have a positive argument in the logs, so there is no need for the absolute value.


x=1/2 gives a negative argument in the log.
Original post by Ano123
x=1/2 gives a negative argument in the log.


Typo, now corrected.
Reply 7
Original post by NotNotBatman
Typo, now corrected.


Does
Unparseable latex formula:

\displaystyle \frac{1}{2} \ln \left \frac{1-x}{x+1}

satisfy the ODE for |x|>1? Is this not a valid solution?
(edited 7 years ago)

Quick Reply

Latest